Question and Answers Forum

All Questions      Topic List

Number Theory Questions

Previous in All Question      Next in All Question      

Previous in Number Theory      Next in Number Theory      

Question Number 13728 by prakash jain last updated on 22/May/17

Prove that  n!>((n/3))^n

$$\mathrm{Prove}\:\mathrm{that} \\ $$ $$\mathrm{n}!>\left(\frac{{n}}{\mathrm{3}}\right)^{{n}} \\ $$

Answered by mrW1 last updated on 23/May/17

Using mathematical induction:  for n=1 we have  1!=1>((1/3))^1 =(1/3) ⇒ true  supposed it′s true for n, i.e.  n!>((n/3))^n   for n+1 we have  (n+1)!=(n+1)n!>(n+1)((n/3))^n   =(((n+1)/3))^(n+1) ((3/(n+1)))((3/(n+1))×(n/3))^n (n+1)  =(((n+1)/3))^(n+1) 3((n/(n+1)))^n >(((n+1)/3))^(n+1)   since 3((n/(n+1)))^n >1  (∗ see proof)  so it′s also true for n+1.  ⇒it′s true for all n.

$${Using}\:{mathematical}\:{induction}: \\ $$ $${for}\:{n}=\mathrm{1}\:{we}\:{have} \\ $$ $$\mathrm{1}!=\mathrm{1}>\left(\frac{\mathrm{1}}{\mathrm{3}}\right)^{\mathrm{1}} =\frac{\mathrm{1}}{\mathrm{3}}\:\Rightarrow\:{true} \\ $$ $${supposed}\:{it}'{s}\:{true}\:{for}\:{n},\:{i}.{e}. \\ $$ $$\mathrm{n}!>\left(\frac{{n}}{\mathrm{3}}\right)^{{n}} \\ $$ $${for}\:{n}+\mathrm{1}\:{we}\:{have} \\ $$ $$\left({n}+\mathrm{1}\right)!=\left({n}+\mathrm{1}\right){n}!>\left({n}+\mathrm{1}\right)\left(\frac{{n}}{\mathrm{3}}\right)^{{n}} \\ $$ $$=\left(\frac{{n}+\mathrm{1}}{\mathrm{3}}\right)^{{n}+\mathrm{1}} \left(\frac{\mathrm{3}}{{n}+\mathrm{1}}\right)\left(\frac{\mathrm{3}}{{n}+\mathrm{1}}×\frac{{n}}{\mathrm{3}}\right)^{{n}} \left({n}+\mathrm{1}\right) \\ $$ $$=\left(\frac{{n}+\mathrm{1}}{\mathrm{3}}\right)^{{n}+\mathrm{1}} \mathrm{3}\left(\frac{{n}}{{n}+\mathrm{1}}\right)^{{n}} >\left(\frac{{n}+\mathrm{1}}{\mathrm{3}}\right)^{{n}+\mathrm{1}} \\ $$ $${since}\:\mathrm{3}\left(\frac{{n}}{{n}+\mathrm{1}}\right)^{{n}} >\mathrm{1}\:\:\left(\ast\:{see}\:{proof}\right) \\ $$ $${so}\:{it}'{s}\:{also}\:{true}\:{for}\:{n}+\mathrm{1}. \\ $$ $$\Rightarrow{it}'{s}\:{true}\:{for}\:{all}\:{n}. \\ $$

Commented byprakash jain last updated on 23/May/17

This is great.   Proof for  3((n/(n+1)))^n >1  We can see that ((n/(n+1)))^n >(((n+1)/(n+2)))^(n+1)  and  lim_(n→∞) ((n/(1+n)))^n =(1/e)  (3/e)>1

$$\mathrm{This}\:\mathrm{is}\:\mathrm{great}.\: \\ $$ $$\mathrm{Proof}\:\mathrm{for} \\ $$ $$\mathrm{3}\left(\frac{{n}}{{n}+\mathrm{1}}\right)^{{n}} >\mathrm{1} \\ $$ $$\mathrm{We}\:\mathrm{can}\:\mathrm{see}\:\mathrm{that}\:\left(\frac{{n}}{{n}+\mathrm{1}}\right)^{{n}} >\left(\frac{{n}+\mathrm{1}}{{n}+\mathrm{2}}\right)^{{n}+\mathrm{1}} \:\mathrm{and} \\ $$ $$\underset{{n}\rightarrow\infty} {\mathrm{lim}}\left(\frac{{n}}{\mathrm{1}+{n}}\right)^{{n}} =\frac{\mathrm{1}}{{e}} \\ $$ $$\frac{\mathrm{3}}{{e}}>\mathrm{1} \\ $$

Commented bymrW1 last updated on 23/May/17

That′s good idea!  I wanted to try to prove 3((n/(n+1)))^n >1 also using  mathematical induction.

$${That}'{s}\:{good}\:{idea}! \\ $$ $${I}\:{wanted}\:{to}\:{try}\:{to}\:{prove}\:\mathrm{3}\left(\frac{{n}}{{n}+\mathrm{1}}\right)^{{n}} >\mathrm{1}\:{also}\:{using} \\ $$ $${mathematical}\:{induction}. \\ $$

Terms of Service

Privacy Policy

Contact: info@tinkutara.com